Những câu hỏi liên quan
Dưa Hấu
Xem chi tiết
Nguyễn Thanh
Xem chi tiết
Đinh Đức Hùng
29 tháng 8 2017 lúc 13:32

Áp dụng Cauchy Schwarz dạng Engel ta có :

\(\frac{a^2}{2a+3b}+\frac{b^2}{2b+3c}+\frac{c^2}{2c+3a}\ge\frac{\left(a+b+c\right)^2}{5\left(a+b+c\right)}=\frac{a+b+c}{5}\)

Dấu "=" xảy ra \(\Leftrightarrow a=b=c\)

Bình luận (0)
Cao Thi Thuy Duong
Xem chi tiết
Nguyễn Việt Lâm
21 tháng 10 2019 lúc 22:53

\(P=\frac{a^2}{2ab+3ac}+\frac{b^2}{2bc+3ab}+\frac{c^2}{2ac+3bc}\)

\(P\ge\frac{\left(a+b+c\right)^2}{5\left(ab+bc+ca\right)}\ge\frac{3\left(ab+bc+ca\right)}{5\left(ab+bc+ca\right)}=\frac{3}{5}\)

Dấu "=" xảy ra khi \(a=b=c\)

Bình luận (0)
 Khách vãng lai đã xóa
Fire Sky
Xem chi tiết
phan tuấn anh
Xem chi tiết
Thắng Nguyễn
1 tháng 4 2017 lúc 21:30

Bài 1:

\(BDT\Leftrightarrow\sqrt{\frac{3}{a+2b}}+\sqrt{\frac{3}{b+2c}}+\sqrt{\frac{3}{c+2a}}\le\frac{1}{\sqrt{a}}+\frac{1}{\sqrt{b}}+\frac{1}{\sqrt{c}}\)

\(\Leftrightarrow\frac{1}{\sqrt{a}}+\frac{1}{\sqrt{b}}+\frac{1}{\sqrt{c}}\ge\sqrt{3}\left(\frac{1}{\sqrt{a+2b}}+\frac{1}{\sqrt{b+2c}}+\frac{1}{\sqrt{c+2a}}\right)\)

Áp dụng BĐT Cauchy-Schwarz và BĐT AM-GM ta có: 

\(\frac{1}{\sqrt{a}}+\frac{1}{\sqrt{b}}+\frac{1}{\sqrt{b}}\ge\frac{9}{\sqrt{a}+\sqrt{2}\cdot\sqrt{2b}}\ge\frac{9}{\sqrt{\left(1+2\right)\left(a+2b\right)}}=\frac{3\sqrt{3}}{\sqrt{a+2b}}\)

Tương tự cho 2 BĐT còn lại ta cũng có: 

\(\frac{1}{\sqrt{b}}+\frac{1}{\sqrt{c}}+\frac{1}{\sqrt{c}}\ge\frac{3\sqrt{3}}{\sqrt{b+2c}};\frac{1}{\sqrt{c}}+\frac{1}{\sqrt{a}}+\frac{1}{\sqrt{a}}\ge\frac{3\sqrt{3}}{\sqrt{c+2a}}\)

Cộng theo vế 3 BĐT trên ta có: 

\(3\left(\frac{1}{\sqrt{a}}+\frac{1}{\sqrt{b}}+\frac{1}{\sqrt{c}}\right)\ge3\sqrt{3}\left(\frac{1}{\sqrt{a+2b}}+\frac{1}{\sqrt{b+2c}}+\frac{1}{\sqrt{c+2a}}\right)\)

\(\Leftrightarrow\frac{1}{\sqrt{a}}+\frac{1}{\sqrt{b}}+\frac{1}{\sqrt{c}}\ge\sqrt{3}\left(\frac{1}{\sqrt{a+2b}}+\frac{1}{\sqrt{b+2c}}+\frac{1}{\sqrt{c+2a}}\right)\)

Đẳng thức xảy ra khi \(a=b=c\)

Bài 2: làm mãi ko ra hình như đề sai, thử a=1/2;b=4;c=1/2

Bình luận (0)
alibaba nguyễn
1 tháng 4 2017 lúc 22:59

Bài 2/

\(\frac{bc}{a^2b+a^2c}+\frac{ca}{b^2c+b^2a}+\frac{ab}{c^2a+c^2b}\)

\(=\frac{b^2c^2}{a^2b^2c+a^2c^2b}+\frac{c^2a^2}{b^2c^2a+b^2a^2c}+\frac{a^2b^2}{c^2a^2b+c^2b^2a}\)

\(=\frac{b^2c^2}{ab+ac}+\frac{c^2a^2}{bc+ba}+\frac{a^2b^2}{ca+cb}\)

\(\ge\frac{\left(bc+ca+ab\right)^2}{2\left(ab+bc+ca\right)}=\frac{ab+bc+ca}{2}\)

\(\ge\frac{3\sqrt[3]{ab.bc.ca}}{2}=\frac{3}{2}\)

Dấu =  xảy ra khi \(a=b=c=1\)

Bình luận (0)
tran thu ha
1 tháng 5 2017 lúc 22:55

bạn alibaba dòng thứ nhất rồi sao ra được dòng thứ hai á bạn mình k hiểu

Bình luận (0)
Y
Xem chi tiết
Nguyễn Việt Lâm
27 tháng 4 2019 lúc 16:16

1.

\(P=\frac{a^4}{abc}+\frac{b^4}{abc}+\frac{c^4}{abc}\ge\frac{\left(a^2+b^2+c^2\right)^2}{3abc}=\frac{\left(a^2+b^2+c^2\right)\left(a^2+b^2+c^2\right)\left(a+b+c\right)}{3abc\left(a+b+c\right)}\)

\(P\ge\frac{\left(a^2+b^2+c^2\right).3\sqrt[3]{a^2b^2c^2}.3\sqrt[3]{abc}}{3abc\left(a+b+c\right)}=\frac{3\left(a^2+b^2+c^2\right)}{a+b+c}\)

Dấu "=" khi \(a=b=c\)

2.

\(P=\sum\frac{a^2}{ab+2ac+3ad}\ge\frac{\left(a+b+c+d\right)^2}{4\left(ab+ac+ad+bc+bd+cd\right)}\ge\frac{\left(a+b+c+d\right)^2}{4.\frac{3}{8}\left(a+b+c+d\right)^2}=\frac{2}{3}\)

Dấu "=" khi \(a=b=c=d\)

Bình luận (0)
Y
27 tháng 4 2019 lúc 14:57

Thục Trinh, tran nguyen bao quan, Phùng Tuệ Minh, Ribi Nkok Ngok, Lê Nguyễn Ngọc Nhi, Tạ Thị Diễm Quỳnh,

Nguyễn Huy Thắng, ?Amanda?, saint suppapong udomkaewkanjana

Help me!

Bình luận (0)
pro
14 tháng 5 2021 lúc 19:47

Bài thứ hai đó áp dụng bđt cauchy showas là ra rồi sử dụng tch bắc cầu tệ.

Bình luận (0)
Aura Phạm
Xem chi tiết
Đinh Đức Hùng
5 tháng 2 2018 lúc 17:42

a) Áp dụng bất đẳng thức AM-GM : 

\(\left(a^2+b^2\right)\left(a^2+1\right)\ge2\sqrt{a^2b^2}.2\sqrt{a^2}\ge2ab.2a=4a^2b\)

b) Áp dụng bất đẳng thức :\(\frac{1}{x}+\frac{1}{y}\ge\frac{4}{x+y}\forall x;y>0\)

 \(\frac{1}{a+3b}+\frac{1}{b+2c+a}\ge\frac{4}{a+3b+b+2c+a}=\frac{4}{2a+4b+2c}=\frac{2}{a+2b+c}\)

Tương tự \(\hept{\begin{cases}\frac{1}{b+3c}+\frac{1}{c+2a+b}\ge\frac{2}{b+2c+a}\\\frac{1}{c+3a}+\frac{1}{a+2b+c}\ge\frac{2}{b+2a+c}\end{cases}}\)

Cộng vế với vế ta được : \(VT+VP\ge2VP\Rightarrow VT\ge VP\)(đpcm)

Bình luận (0)
Trần Lâm Thiên Hương
Xem chi tiết
Trần Lâm Thiên Hương
15 tháng 5 2018 lúc 21:03

Mình nhầm, phải là \(\le\frac{1}{3}\)mọi người làm giúp mình với mình cần gấp

Bình luận (0)
zZz Cool Kid_new zZz
1 tháng 8 2020 lúc 19:31

Theo BĐT Cauchy Schwarz và các biến đổi cơ bản ta dễ có được:
\(\frac{a^2}{\left(2a+b\right)\left(2a+c\right)}=\frac{a^2}{2a\left(a+b+c\right)+2a^2+bc}=\frac{1}{9}\left[\frac{\left(2a+a\right)^2}{2a\left(a+b+c\right)+2a^2+bc}\right]\)

\(\le\frac{1}{9}\left[\frac{4a^2}{2a\left(a+b+c\right)}+\frac{a^2}{2a^2+bc}\right]=\frac{1}{9}\left(\frac{2a}{a+b+c}+\frac{a^2}{2a^2+bc}\right)\)

\(\Rightarrow LHS\le\frac{1}{9}\left(2+\frac{a^2}{2a^2+bc}+\frac{b^2}{2b^2+ca}+\frac{c^2}{2c^2+ab}\right)\)

Tiếp tục theo BĐT Cauchy Schwarz dạng Engel:

\(\frac{a^2}{a^2+2bc}+\frac{b^2}{b^2+2ca}+\frac{c^2}{c^2+2ab}\ge\frac{\left(a+b+c\right)^2}{\left(a+b+c\right)^2}=1\)

Ta thực hiện phép đổi biến thì:

\(\frac{ab}{ab+2c^2}+\frac{bc}{bc+2a^2}+\frac{ca}{ca+2b^2}\ge1\)

Đến đây là phần của bạn

Bình luận (0)
 Khách vãng lai đã xóa
tth_new
3 tháng 8 2020 lúc 19:10

(Vào thống kê hỏi đáp xem ảnh nhé! 2 cách, cách đầu dùng kỹ thuật uvw, cách kia là SOS)

Bình luận (0)
 Khách vãng lai đã xóa
Lê Minh Đức
Xem chi tiết
Akai Haruma
9 tháng 10 2017 lúc 17:28

Ta có \(a^2b^2+b^2c^2+c^2a^2\geq a^2b^2c^2\Leftrightarrow \frac{1}{a^2}+\frac{1}{b^2}+\frac{1}{c^2}\geq 1\)

BĐT cần chứng minh tương đương với \(\frac{\frac{1}{c^3}}{\frac{1}{a^2}+\frac{1}{b^2}}+\frac{\frac{1}{b^3}}{\frac{1}{a^2}+\frac{1}{c^2}}+\frac{\frac{1}{a^3}}{\frac{1}{b^2}+\frac{1}{c^2}}\geq \frac{\sqrt{3}}{2}\)

Đặt \((\frac{1}{a},\frac{1}{b},\frac{1}{c})=(x,y,z)\). Bài toán trở thành: 

Cho \(x,y,z>0|x^2+y^2+z^2\geq 1\). CMR \(P=\frac{x^3}{y^2+z^2}+\frac{y^3}{z^2+x^2}+\frac{z^3}{x^2+y^2}\geq \frac{\sqrt{3}}{2}\)

Lời giải:

 Áp dụng BĐT Cauchy -Schwarz:

\(P=\frac{x^4}{xy^2+xz^2}+\frac{y^4}{yz^2+yx^2}+\frac{z^4}{zx^2+zy^2}\geq \frac{(x^2+y^2+^2)^2}{x^2(y+z)+y^2(x+z)+z^2(x+y)}\) (1)

Không mất tính tổng quát, giả sử \(x\geq y\geq z\Rightarrow x^2\geq y^2\geq z^2\) 

Và \(y+z\leq z+x\leq x+y\). Khi đó, áp dụng BĐT Chebyshev: 

\(3[x^2(y+z)+y^2(x+z)+z^2(x+y)]\leq (x^2+y^2+z^2)(y+z+x+z+x+y)\)

\(\Leftrightarrow x^2(y+z)+y^2(x+z)+z^2(x+y)\leq \frac{2(x^2+y^2+z^2)(x+y+z)}{3}\)

Theo hệ quả của BĐT Am-Gm thì: \((x+y+z)^2\leq 3(x^2+y^2+z^2)\Rightarrow x+y+z\leq \sqrt{3(x^2+y^2+z^2)}\)

\(\Rightarrow x^2(y+z)+y^2(x+z)+z^2(x+y)\leq \frac{2(x^2+y^2+z^2)\sqrt{3(x^2+y^2+z^2)}}{3}\) (2)

Từ (1),(2) suy ra \(P\geq \frac{3(x^2+y^2+z^2)^2}{2(x^2+y^2+z^2)\sqrt{3(x^2+y^2+z^2)}}=\frac{\sqrt{3(x^2+y^2+z^2)}}{2}\geq \frac{\sqrt{3}}{2}\)

Ta có đpcm

Dáu bằng xảy ra khi \(x=y=z=\frac{1}{\sqrt{3}}\Leftrightarrow a=b=c=\sqrt{3}\)

Bình luận (0)
Kiệt Nguyễn
5 tháng 5 2020 lúc 12:58

Đặt \(x=\frac{1}{a};y=\frac{1}{b};z=\frac{1}{c}\)

Khi đó giả thiết được viết lại là \(x^2+y^2+z^2\ge1\)và ta cần chứng minh \(\frac{x^3}{y^2+z^2}+\frac{y^3}{z^2+x^2}+\frac{z^3}{x^2+y^2}\ge\frac{\sqrt{3}}{2}\)(*)

Áp dụng BĐT Bunhiacopxki dạng phân thức, ta được:

\(VT_{\left(^∗\right)}=\frac{x^4}{x\left(y^2+z^2\right)}+\frac{y^4}{y\left(z^2+x^2\right)}+\frac{z^4}{z\left(x^2+y^2\right)}\)\(\ge\frac{\left(x^2+y^2+z^2\right)^2}{x\left(y^2+z^2\right)+y\left(z^2+x^2\right)+z\left(x^2+y^2\right)}\)

Đến đây ta đi chứng minh \(\frac{\left(x^2+y^2+z^2\right)^2}{x\left(y^2+z^2\right)+y\left(z^2+x^2\right)+z\left(x^2+y^2\right)}\ge\frac{\sqrt{3}}{2}\)

\(\Leftrightarrow2\left(x^2+y^2+z^2\right)^2\)\(\ge\sqrt{3}\left[x\left(y^2+z^2\right)+y\left(z^2+x^2\right)+z\left(x^2+y^2\right)\right]\)

Ta có: \(x\left(y^2+z^2\right)=\frac{1}{\sqrt{2}}\sqrt{2x^2\left(y^2+z^2\right)\left(y^2+z^2\right)}\)\(\le\frac{1}{\sqrt{2}}\sqrt{\left(\frac{2x^2+y^2+z^2+y^2+z^2}{3}\right)^3}\)

\(=\frac{2\sqrt{3}}{9}\left(x^2+y^2+z^2\right)\sqrt{x^2+y^2+z^2}\)

Tương tự ta có: \(y\left(z^2+x^2\right)\le\frac{2\sqrt{3}}{9}\left(x^2+y^2+z^2\right)\sqrt{x^2+y^2+z^2}\)

\(z\left(x^2+y^2\right)\le\frac{2\sqrt{3}}{9}\left(x^2+y^2+z^2\right)\sqrt{x^2+y^2+z^2}\)

Cộng theo vế của 3 BĐT trên, ta được: 

\(\text{∑}_{cyc}\left[x\left(y^2+z^2\right)\right]\le\frac{2\sqrt{3}}{3}\left(x^2+y^2+z^2\right)\sqrt{x^2+y^2+z^2}\)

\(\Leftrightarrow\sqrt{3}\text{∑}_{cyc}\left[x\left(y^2+z^2\right)\right]\le2\left(x^2+y^2+z^2\right)\sqrt{x^2+y^2+z^2}\)

Cuối cùng ta cần chứng minh được

\(2\left(x^2+y^2+z^2\right)\sqrt{x^2+y^2+z^2}\le2\left(x^2+y^2+z^2\right)^2\)

\(\Leftrightarrow x^2+y^2+z^2\ge1\)(đúng)

Đẳng thức xảy ra khi \(x=y=z=\frac{1}{\sqrt{3}}\Rightarrow a=b=c=\sqrt{3}\)

Bình luận (0)
 Khách vãng lai đã xóa